LSAT and Law School Admissions Forum

Get expert LSAT preparation and law school admissions advice from PowerScore Test Preparation.

 James Finch
PowerScore Staff
  • PowerScore Staff
  • Posts: 943
  • Joined: Sep 06, 2017
|
#73401
Hi Kalifa,

Taking a contrapositive of a conditional chain is the same as taking a single conditional's contrapositive: you flip and negate the entire chain. So if we have a chain of:

GHBT :arrow: PRZA :arrow: ABW&SS :arrow: BNR :arrow: STI

then our contrapositive chain would look like:

STI :arrow: BNR :arrow: ABW&SS :arrow: PRZA :arrow: GHBT

Using the chains, we could also make inferences such as:

GHBT :arrow: STI

and

STI :arrow: GHBT

Hope this clears things up!
User avatar
 christinecwt
  • Posts: 74
  • Joined: May 09, 2022
|
#95231
Hi All - just wonder why can't choose the answer D instead of answer B....Many thanks!
 Adam Tyson
PowerScore Staff
  • PowerScore Staff
  • Posts: 5153
  • Joined: Apr 14, 2011
|
#95242
Answer D is a Mistaken Negation, christinecwt - we know that building new apartments would be sufficient to make higher taxes necessary, but we can never say that the absence of a sufficient condition (no new apartments) would prove the absence of the necessary condition (no higher taxes). Taxes might still go up for any number of other reasons! The necessary condition in a conditional relationship can occur even when the sufficient condition does not. For example:

If I eat this whole giant box of cookies, I will make myself sick.

I didn't eat the whole box.

Does this prove I didn't make myself sick? Nope. There's still a lot of chips and ice cream in the house that I could gorge on!
User avatar
 christinecwt
  • Posts: 74
  • Joined: May 09, 2022
|
#95259
Thanks Adam! This helps a lot! :)
User avatar
 ericsilvagomez
  • Posts: 49
  • Joined: Oct 16, 2023
|
#104358
Hi,

I chose answer choice A, and I think I got it wrong for two reasons: the "will" in the answer choice is too strong, and the first sentence says that unless they band together the proposal will be approved, not that developers would not build apartment houses. Without writing out the conditional relationships, I think B could be correct because it seems to connect with the different relationships throughout the stimulus. As for my conditional relationships, I put: "proposal to rezone the city will be approved" -> "the residents of Glen Hills band together" -> "that developers need in order to construct apartment houses here" -> "roads (crossed out) or additional schools (crossed out)"-> "substantial tax increases for the residents of Glen Hills" What do you think of this outline of the conditional relationships? Also, is the correct answer choice supposed to connect to some of them, like I mentioned when describing choosing B?
User avatar
 srusty
PowerScore Staff
  • PowerScore Staff
  • Posts: 32
  • Joined: Nov 30, 2023
|
#104406
ericsilvagomez wrote: Fri Dec 08, 2023 7:43 am Hi,

I chose answer choice A, and I think I got it wrong for two reasons: the "will" in the answer choice is too strong, and the first sentence says that unless they band together the proposal will be approved, not that developers would not build apartment houses. Without writing out the conditional relationships, I think B could be correct because it seems to connect with the different relationships throughout the stimulus. As for my conditional relationships, I put: "proposal to rezone the city will be approved" -> "the residents of Glen Hills band together" -> "that developers need in order to construct apartment houses here" -> "roads (crossed out) or additional schools (crossed out)"-> "substantial tax increases for the residents of Glen Hills" What do you think of this outline of the conditional relationships? Also, is the correct answer choice supposed to connect to some of them, like I mentioned when describing choosing B?
Hi Eric,

You're right about answer choice (A), the "will" is too strong. With the conditional reasoning, the "unless" indicates a necessary condition, as such:
~Proposal Approved -----> Residents Banded Together

To get to the correct answer, for this question, it is not necessary to go through the conditional chains. The statements in the stimulus lay out for us that if new apartments are build, they will have increased population, needing new roads, and for the new roads/schools, we will need to tax Glen Hills residents. However, to organize the statements, it can be helpful to link the concepts together with conditional reasoning.

Hope this helps!
User avatar
 askuwheteau@protonmail.com
  • Posts: 17
  • Joined: Feb 08, 2024
|
#105244
This problem challenged me. I diagramed the stimulus: Don't band together>proposal approved>apts built>new residents>overcrowding of schools and roads> new roads built>resid tax increases.

Here's my logic:

A: is a mistaken negation of a portion of the conditional chain
B: is a repeat of a portion of the conditional chain
C: is Out of Scope with the conditional chain (the chain doesn't discuss loss of rural atmosphere)
D: is a mistaken negation of a portion of the conditional chain
E: is a mistaken negation of a portion of the conditional chain

Am I correct in my understanding of why B is the correct answer and A, C, D, &, E are wrong answers.

Thanks,

Jonathan
User avatar
 Chandler H
PowerScore Staff
  • PowerScore Staff
  • Posts: 54
  • Joined: Feb 09, 2024
|
#105253
Hi Jonathan,

This is a pretty challenging one! You're 90% of the way there in your understanding of why answer choice (B) is correct, but there is one key component you might be missing.

The stimulus tells us that, if the proposal is approved, "... the city will be able to build the water and sewer systems that developers need in order to construct apartment houses there. These buildings would attract new residents..."

If we read that closely, we see that the conditional expressed here is as follows:

CAH :arrow: BW&SS

In other words, it is necessary to build the water and sewer systems in order to construct the apartment houses. It would be a mistaken reversal to say that BW&SS :arrow: CAH. Just because the water and sewer systems are built doesn't mean that the apartment houses will 100% be built.

However, answer choice (B) is valid because, like you said, it restates the portion of the conditional chain AFTER the new apartments are built, which looks like this:

CAH :arrow: ANR :arrow: NRB :arrow: STI

That is, if those apartment houses are constructed, then they'll attract new residents, and then new roads will be built, and then there will be substantial tax increases. Therefore, answer choice (B) is correct.

Hopefully this all makes sense!
User avatar
 askuwheteau@protonmail.com
  • Posts: 17
  • Joined: Feb 08, 2024
|
#105291
Chandler H wrote: Fri Feb 09, 2024 1:20 pm Hi Chandler,

I went back and reread the stimulus and see that I missed the sufficient condition indicator (in order to). This certainly was a subtle, but important point which strengthened by understanding of this conditional reasoning-heavy type problem. Thank you for taking time to address my question.

Jonathan

Get the most out of your LSAT Prep Plus subscription.

Analyze and track your performance with our Testing and Analytics Package.